How Does the Cancellation Property Prove Divisibility in Z?

Click For Summary
The discussion centers on proving the cancellation property in the integers, specifically that for any integers a, b, and non-zero c, a divides b if and only if ca divides cb. The key point is understanding that if ca divides cb, one can apply the cancellation property to deduce that a divides b, provided c is not zero. The user expresses confusion about formalizing this proof, particularly in relation to the cancellation property. They attempt to outline a proof but feel it lacks depth and clarity regarding the cancellation property. Overall, the conversation emphasizes the importance of clearly demonstrating the cancellation property in the proof process.
mikky05v
Messages
53
Reaction score
0
Homework Statement


this is the original question
prove: \forall c \in Z, a≠ 0 and b both \in Z$
a|b ⇔ c*a|c*b

Then he corrected himself by saying for problem 1: to show that ca | cb implies a | b ... you must assume c NOT = 0 and invoke "Cancellation Property" of Z.

This kind of confused me but i think I get what he means

The attempt at a solution

so I understand that If you have that ca | cb that's like saying that ac=cbq for some q∈ℤ so, if c≠0 you can just take out those c in the both sides of the expression(because of "Cancellation Property" as he said) and you got left a=bq which means that a|b

my problem is how do I translate this into a formal proof if and only if proof.
 
Physics news on Phys.org
ok so this is what I've got
Prove: ∀c∈Z, c≠0 and b both∈Z a|b⇔ca|cb
a|b if and only if b=ak for some k∈Z
if and only if cb=cak for some c∈Z
if and only if ac|cb

Is this a valid proof? It seems kind of short and it's lacking the "cancelation property" but I'm not sure I understand how to write it any other way
 
Question: A clock's minute hand has length 4 and its hour hand has length 3. What is the distance between the tips at the moment when it is increasing most rapidly?(Putnam Exam Question) Answer: Making assumption that both the hands moves at constant angular velocities, the answer is ## \sqrt{7} .## But don't you think this assumption is somewhat doubtful and wrong?

Similar threads

  • · Replies 2 ·
Replies
2
Views
2K
Replies
2
Views
1K
Replies
3
Views
2K
  • · Replies 1 ·
Replies
1
Views
1K
Replies
1
Views
1K
  • · Replies 4 ·
Replies
4
Views
2K
  • · Replies 26 ·
Replies
26
Views
3K
  • · Replies 4 ·
Replies
4
Views
2K
  • · Replies 2 ·
Replies
2
Views
2K
  • · Replies 3 ·
Replies
3
Views
1K